Stokes Theorem: Vector Integral Identity Proof

In summary, the author is trying to solve the equation ##\vec B\cdot \left(\oint_C \psi \ dr\right)=B\cdot\left(\iint_S (\hat{n}\times\nabla\psi) dS\right)##, but they believe they made a mistake.
  • #1
Master1022
611
117
TL;DR Summary
It is a proof question, but I am struggling to understand what steps I should take next
Hi,

My question pertains to the question in the image attached.
Screen Shot 2020-01-15 at 3.56.25 PM.png


My current method:
Part (a) of the question was to state what Stokes' theorem was, so I am assuming that this part is using Stokes' Theorem in some way, but I fail to see all the steps.

I noted that [tex] \nabla \times \vec F = \nabla \times (\psi \vec B) = \psi \left( \nabla \times \vec B \right) - \vec B \times \nabla \psi [/tex]
Then I thought that [itex] \nabla \times \vec B = \vec 0 [/itex] as [itex] \vec B [/itex] is a constant vector. Simplifying the above expression, we are left with:
[tex] \nabla \times \vec F = - \vec B \times \nabla \psi [/tex] Then I attempted to include the normal vector [itex] \hat n dS [/itex] by 'dotting' both sides with it:
[tex] \left( \nabla \times \vec F \right) \cdot \hat n dS = \left( - \vec B \times \nabla \psi \right) \cdot \hat n dS [/tex]
Then, using Stokes Theorem, integrating both sides over the surface S, and swapping the LHS for the line integral and using the scalar triple product identity for the RHS:
[tex] \left( - \vec B \times \nabla \psi \right) \cdot \hat n dS = \left(\nabla \psi \times \vec B \right) \cdot \hat n dS = \left(\hat n \times \nabla \psi \right) \cdot \vec B dS [/tex]
[tex] \oint \left( \psi \vec B \right) \cdot d\vec r = \iint_S \left(\hat n \times \nabla \psi \right) \cdot \vec B dS [/tex]

What I have there doesn't look too different from the required solution, however I believe that I may have made a grave error along the way. Would anyone be able to provide me with some suggestions/hints on how to proceed?

Thanks in advance
 
Physics news on Phys.org
  • #2
You can re-write your last line as ##B\cdot \left(\oint_C \psi \ dr\right)=B\cdot\left(\iint_S (\hat{n}\times\nabla\psi) dS\right)##. In general if ##v## and ##w## are vectors and ##B\cdot v=B\cdot w## for all vectors ##B##, then ##v## must equal ##w##.
 
  • Like
Likes Master1022
  • #3
Infrared said:
You can re-write your last line as ##B\cdot \left(\oint_C \psi \ dr\right)=B\cdot\left(\iint_S (\hat{n}\times\nabla\psi) dS\right)##. In general if ##v## and ##w## are vectors and ##B\cdot v=B\cdot w## for all vectors ##B##, then ##v## must equal ##w##.

Many thanks. Yes, that makes sense.
 

1. What is Stokes Theorem?

Stokes Theorem is a mathematical theorem that relates the surface integral of a vector field over a closed surface to the line integral of the same vector field around the boundary of that surface.

2. What is the Vector Integral Identity Proof?

The Vector Integral Identity Proof is a mathematical proof that shows the equivalence between the surface integral and line integral in Stokes Theorem. It is a key step in understanding and applying Stokes Theorem in various mathematical and scientific fields.

3. How is Stokes Theorem used in science?

Stokes Theorem is used in many fields of science, including physics, engineering, and mathematics. It is particularly useful in the study of fluid dynamics, electromagnetism, and other areas where vector fields are present.

4. What are the assumptions of Stokes Theorem?

Stokes Theorem makes two main assumptions: the surface over which the integral is being evaluated must be closed (meaning it has a defined boundary), and the vector field being integrated must be continuous and differentiable within the region enclosed by the surface.

5. Can Stokes Theorem be extended to higher dimensions?

Yes, Stokes Theorem can be extended to higher dimensions through the use of differential forms and the generalization of the surface integral to higher-dimensional manifolds. This extension is known as the Generalized Stokes Theorem or the Stokes-Cartan Theorem.

Similar threads

Replies
5
Views
1K
  • Calculus
Replies
4
Views
1K
Replies
8
Views
2K
  • Introductory Physics Homework Help
Replies
1
Views
103
  • Introductory Physics Homework Help
Replies
3
Views
327
Replies
2
Views
1K
  • Calculus and Beyond Homework Help
Replies
9
Views
753
  • Calculus
Replies
8
Views
2K
Replies
1
Views
351
Back
Top